(Ex) 3 (1) .1-3.2
(Ex) 3 (1) .1-3.2
1 (Selected Problems)
1. For the mapping w = (1 + i)z + 2, find the image of
(b) the line y = 3x + 2
Solution. Let w = u + iv and z = x + iy for convenience. Then w = (1 + i)z + 2 if and only
if u + iv = (1 + i)(x + iy) + 2. Namely,
1 1
x= (u + v) − 1, y= (v − u) + 1.
2 2
Therefore
1 3
y = 3x + 2 ⇔ (v − u) + 1 = (u + v) − 1 ⇔ 2u + v − 2 = 0.
2 2
(d) the circle |z − 1| = 2
w−2
Solution. Since z = , |z − 1| = 2 becomes
1+i
¯w − 2 ¯ |w − (3 + i)|
¯ ¯
¯ − 1¯ = 2, namely, = 2.
1+i |1 + i|
√
Therefore the image is a circle of radius 2 2 centered at 3 + i. ¤
2. Find the image of the half plane Re z > 0 under the transformation
(a) w = 2iz − i
Solution. If w = u + iv (u, v ∈ R) then Re(iw) = Re(iu − v) = −v = −Im w. Since
w+i iw 1
z= =− + ,
2i 2 2
³ iw ´ 1 1 1 1 1
Re z = Re − + = − Re (iw) + = Im w + .
2 2 2 2 2 2
Thus the image is the half plane {w | Im w > −1} = {(x, y) | y > −1}. ¤
Alternate approach: Note that w = 2iz − i = (f3 ◦ f2 ◦ f1 )(z), where f1 (z) = iz, f2 (z) = 2z
and f3 (z) = z − i.
The rotation f1 maps the right half plane {z | Re z > 0} onto the upper half plane {z |
Im z > 0}. The dilation f2 maps {z | Im z > 0} onto itself.
Finally the translation f3 maps {z | Im z > 0} onto the half plane {w | Im w > −1}. ////
i
(b) w = −1
z
i iw + i
Solution. If we let z = x + iy and w = u + iv for convenience, Then z = = .
w+1 |w + 1|2
v + i(u + 1)
Then x + iy = , namely,
(u + 1)2 + v 2
v u+1
x= , y= .
(u + 1)2 + v 2 (u + 1)2 + v 2
Therefore x > 0 if and only if v > 0. The image is the half plane {(u, v) | v > 0} = {w |
Im w > −1}. ¤
1
3. Find the image of the semi-infinite strip 0 < Re z < 2, Im z > 1 for the transformation
w = (1 − i)z + (2 − i).
Solution. Sometimes it is convenient to use the rectangular representation for z and w. Let
w−2+i 1
z = x + iy and w = u + iv. Then it follows that z = = ((1 + i)w − 3 − i). In
1−i 2
other words,
1 1
x = Re z = (u − v − 3), y = Im z = (u + v − 1).
2 2
Therefore 0 < Re z < 2 if and only if 3 < u − v < 7. Moreover Im z > 1 if and only if
u + v > 3. The image is {(u, v) | 3 < u − v < 7 and u + v > 3}.
6. Given a triangle with vertices at 3 + 4i, −3 + 4i and −5i, find its image for the transformation
f (z) = az + b (a 6= 0)
Solution. Note that the linear function f (z) = az + b (a 6= 0) maps a straight line onto a
straight line. As a subset of a straight line, every line segment is mapped to a line segment
under f (z): if we let A, B, C be the three vertices of a given triangle, then the line segment
AB is mapped onto the line segment joining f (A) to f (B), and so on. Therefore the image
of the triangle is a triangle with vertices f (3 + 4i), f (−3 + 4i) and f (−5i). ¤
(b) w = i/z
Solution. We may use the result in (a). Since the function f (z) = i/z is a composite
function of f1 (z) and ¯f2 (z), where¯ f1 (z)
√
= iz and f2 (z) = 1/z. All we have to do is to
¯ 1 ¯ 5
rotate the circle {z | ¯z + 2 (2 + i)¯ = 2 } by the angle π/2 (why?). Thus the image is
¯ ¯ √
¯ ¯
{w | ¯w + 12 (−1 + 2i)¯ = 25 }.
2
2 2 2
Therefore y = 2x + 1 if and only if u + 2u¯ + 2v = 2v +¯ u √ + v 2 . The image is a circle
³ ´2
¯ ¯
{(u, v) | u + 12 + (v − 1)2 = 45 } = {w | ¯w − 12 (−1 + 2i)¯ = 25 }. ¤
11. Find the images of the strips 0 ≤ Re z ≤ 2 and 0 ≤ Im z ≤ 2 under the map w = 1/z.
Solution. Use the argument in #8-(e). We skip the details.
3
iz
2. Under the transformation w = , find the image of
z−1
(a) the unit disk |z| ≤ 1.
i
Solution. Since z = 1 + , it follows that
w−i
¯ i ¯¯ |w|
¯
1 ≥ |z| = ¯1 + ¯= ,
w−i |w − i|
namely, |w| ≤ |w − i|. The image is the half plane {(x, y) | y ≤ 1/2}. ////
Method 2: Choose three distinct points 1, −1, i on the unit circle |z| = 1, which is the
boundary of |z| ≤ 1. These three points are mapped to ∞, i/2 and (1 + i)/2, respectively.
Since the image of |z| = 1 contains ∞, it is a straight line passing through the points
i/2 and (1 + i)/2, namely, {w | Im z = 1/2}. Therefore the image of the unit disk is
{w | Im w ≤ 1/2} (why?).
Here we used the fact that every LFT maps circles and straight lines onto circles or straight
lines. ////
π π
(d) the infinite strip< Arg z < .
4 2
Solution. We can use the rectangular representation for z = x + iy and w = u + iv. Since
i i(w + i)
z =1+ =1+ ,
w−i |w − i|2
v−1 u
x=1+ , y= . (2)
u2 + (v − 1)2 u2 + (v − 1)2
π π
On the other hand, < Arg z < if and only if y > x and x > 0. Note that
4 2
√
y > x ⇔ u2 + v 2 − u − v < 0 ⇔ |w − (1 + i)/2| < 1/ 2
and
x>0 ⇔ u2 + v 2 − v > 0 ⇔ |w − (i/2)| > 1/2.
////
Method 2: Arg z = π/2 is a ray, and it is a part of the imaginary axis. Choose three distinct
points 0, i and ∞ on the imaginary axis. Under the transformation f (z) = iz/(z − 1), three
points 0, i, ∞ are mapped to 0, (1 + i)/2, i respectively, which are located on the circle
C1 : {w | |w − (i/2)| = 1/2}. Thus the image of Arg = π/2 is a subset of C1 .
Consider a particle P on the ray Arg = π/2 moving toward the origin. If P is far away
from the origin, f (P ) is sufficiently close to i. If P is sufficiently close to the origin, then
f (P ) is sufficiently close to 0. Consequently, as P moves towards the origin f (P ) moves to
0 along the circle C1 . Therefore the image of Arg = π/2 is the set C1 ∩ {w | Re w > 0}.
The ray Arg = π/4 is a part of a straight line y = x. The three points 0, 1+i, ∞ are mapped
√
to 0, 1 + i, i respectively, which are located on the circle C2 : {w | |w − (1 + i)/2| = 1/ 2}.
4
It is easy to check that the image of Arg = π/4 is the set C2 ∩ {w | Re w > 0}. (Draw a
picture!)
√
Therefore, we conclude that {w | |w − (i/2)| > 1/2 and |w − (1 + i)/2| < 1/ 2} is the
image. (why?) ////
Remark. You can also use (2) to solve (b) and (c).
z−1
3. Under the transformation w = , find the image of
z+1
(a), (b) |z| ≤ r (r 6= 1)
1+w
Solution. Note that z = . Then |z| ≤ r if and only if |w + 1| ≤ r|w − 1|. The
1−w
¯ 1 + r2 ¯¯ 2r
¯
boundary |w + 1| = r|w − 1| is indeed the circle ¯w + ¯= .
1 − r2 |1 − r2 |
Note that the center w0 = −(1+r2 )/(1−r2 ) satisfies |w0 +1| < r|w0 −1| if r < 1. Moreover
|w0 + 1| > r|w0 − 1| if r > 1. Therefore
¯ 1 + r2 ¯¯ 2r ¯ 1 + r2 ¯¯ 2r
¯ ¯
¯w + 2 ¯≤ (r < 1), ¯w + ¯≥ (r > 1)
1−r |1 − r2 | 1−r 2 |1 − r2 |
(c) (Method 2): The boundary of {z | Im z > 1} is a straight line {z | Im z = 1}. Choose
three points i, −1 + i and ∞ on this line. Under the transformation w = (z − 1)/(z + 1),
these points are mapped to i, 1 + 2i and 1 respectively, which are located on the circle
|w − (1 + i)| = 1.
Then the image is the disk {w | |w − (1 + i)| < 1} (why?). ////
14. Find all linear fractional transformations mapping the imaginary axis onto the unit circle.
Solution.
(Method 1) Let ϕ(z) = −iz. Then ϕ denotes a rotation by the angle −π/2, and it maps
the imaginary axis onto the real axis. Note that ϕ−1 (z) = iz.
If f (z) is a desired LFT, then f ◦ ϕ−1 is an LFT mapping the real axis onto the unit circle.
Then it follows that
z − z0
(f ◦ ϕ−1 )(z) = A
z − z0
5
for some constants A and z0 such that |A| = 1 and Im z0 6= 0.
−iz − z0 z − iz0
Therefore f (z) = A =A , (|A| = 1, Im z0 6= 0). ¤
−iz − z 0 z − iz 0
az + b
(Method 2: Direct approach) Assume that f (z) is expressed in the form f (z) = .
cz + d
Choose three points 0, i, ∞ in (the imaginary axis) ∪{∞}. Since f (0) = b/d and f (∞) =
a/c, it follows that
|b| |a|
=1= .
|d| |c|
In particular, |b/a| = |d/c|. Let A = a/c, z1 = −b/a and z2 = −d/c. It is easily checked
that z1 6= z2 , |z1 | = |z2 | and
a z + b/a z − z1
f (z) = · =A .
c z + d/c z − z2
i − z1
We claim that Im z1 6= Im z2 . Indeed, since f (i) = A belongs to the unit circle
i − z2
{w | |w| = 1}, it follows that |z1 − i| = |z2 − i|. Since |z1 | = |z2 |, we conclude that
Im z1 = Im z2 .
z − z1
(Conclusion) f (z) = A for some constants z1 , z2 (z1 6= z2 ) and A such that |A| = 1
z − z2
and Im z1 = Im z2 . ////
Remark. In this case, z1 = iz0 and z2 = iz 0 for some constant z0 such that Im z0 6= 0.
15. Show that a linear fractional transformation that maps the disk |z| ≤ r1 onto the disk |w| ≤ r2
r1 r2 (z − z0 )
must be of the form w = A 2 , where |A| = 1 and |z0 | < r1 .
r1 − z 0 z
Solution. Let ϕ1 (z) = z/r1 and ϕ2 (z) = z/r2 . Then ϕ1 (z) (ϕ2 (z), respectively) maps the
disk |z| ≤ r1 (|z| ≤ r2 , respectively) onto the unit disk |z| ≤ 1.
If f is a desired LFT, then ϕ2 ◦ f ◦ ϕ−1
1 maps the unit disk onto the unit disk. Then it
−1 z−γ
follows that (ϕ2 ◦ f ◦ ϕ1 )(z) = K := w∗ (z) with |K| = 1 and |γ| < 1. Therefore
γz − 1
z − r1 γ r1 r2 (r1 γ − z)
f (z) = (ϕ−1 ∗ ∗
2 ◦ w ◦ ϕ1 )(z) = r2 w (z/r1 ) = r2 K =K 2 .
γz − r1 r1 − r1 γz